Compact Nested Sequences and Their Intersection

  • Thread starter Thread starter JamesF
  • Start date Start date
  • Tags Tags
    Compact Sequences
Click For Summary
The discussion revolves around proving that the intersection of a sequence of compact, nonempty subsets of a metric space is non-empty. The user attempts a proof by contradiction, assuming the intersection is empty and analyzing the open cover formed by the complements of the sets. They realize that this leads to a contradiction regarding the compactness of the sets involved. Additionally, the user inquires whether the set A_n = [n, ∞) is open or closed in ℝ, concluding that it is closed despite being unbounded. The thread highlights the importance of understanding compactness in metric spaces and the properties of open and closed sets.
JamesF
Messages
13
Reaction score
0
Hi everyone. I feel like I'm really close to the answer on this one, but just out of reach :) I hope someone can give me some pointers

Homework Statement



Let A1 \supseteq A2 \supseteq A3 \supseteq \ldots be a sequence of compact, nonempty subsets of a metric space (X, d). Show that \bigcap A_n \neq \emptyset. (Hint: Let U_n = X-A_n)

The Attempt at a Solution


I tried showing by contradiction.

Suppose \bigcap A_n = \emptyset
Choose an open subcover U_n = X-A_n (that's supposed to be set minus but I don't know how to do \ in tex). Then \bigcup U_n = (X-A_1) \cup (X - A_2) \ldots = X - (\bigcap A_n) = X

but where's the contradiction? So X is not compact, but that goes without saying and we can't infer much from that. What am I overlooking here? Or is this the wrong approach entirely?

Thank you for your assistance.
 
Physics news on Phys.org
X may not be compact but A1 is. The U_n are a cover of A1 if you assume the intersection of all the Ai is empty. Hence there is a finite subcover. That seems headed for a contradiction.
 
Thanks for your help, Dick. I was able to get the solution.


I have one more question on my current HW.
Is a set A_n = [n, \infty) open or closed in \mathbb{R}? I would think so, but it's unbounded.
 
JamesF said:
Thanks for your help, Dick. I was able to get the solution.


I have one more question on my current HW.
Is a set A_n = [n, \infty) open or closed in \mathbb{R}? I would think so, but it's unbounded.

What do you think it would be?
 
Question: A clock's minute hand has length 4 and its hour hand has length 3. What is the distance between the tips at the moment when it is increasing most rapidly?(Putnam Exam Question) Answer: Making assumption that both the hands moves at constant angular velocities, the answer is ## \sqrt{7} .## But don't you think this assumption is somewhat doubtful and wrong?

Similar threads

  • · Replies 1 ·
Replies
1
Views
2K
Replies
4
Views
2K
  • · Replies 8 ·
Replies
8
Views
2K
Replies
11
Views
3K
  • · Replies 7 ·
Replies
7
Views
2K
  • · Replies 14 ·
Replies
14
Views
3K
  • · Replies 11 ·
Replies
11
Views
2K
  • · Replies 5 ·
Replies
5
Views
2K
  • · Replies 8 ·
Replies
8
Views
2K
  • · Replies 5 ·
Replies
5
Views
11K